¿Por qué no se ha mostrado el monopolo magnético en el contexto del modelo estándar de la física de partículas?

Por diversas razones, los expertos, desde Dirac , creen que una teoría del campo unificado debe tener un monopolo magnético . Por ejemplo, Polyakov dijo: "Estoy bastante seguro de que los monopolos magnéticos realmente existen. Cómo, cuándo y si se encontrarán es otra cuestión".

Extrañamente, pensando que tantas partículas elementales en el modelo estándar, no tenemos ninguna partícula que lleve una carga de monopolo magnético U (1), que debería existir para una teoría de calibre Abelian U (1) compacta. Por ejemplo, los quarks y los leptones pueden llevar varias cargas de la U(1) eléctrica mi metro y la carga débil SU(2) w mi a k ; y los quarks también llevan la carga fuerte, la SU(3) s t r o norte gramo colores. Una vez más, todavía no tenemos ninguna partícula elemental que lleve la carga del monopolo magnético U(1).

Para un propósito teórico, ¿es posible tener una partícula elemental como quarks y leptones en el modelo estándar unida a un monopolo magnético U(1)?

En caso afirmativo, ¿cómo podemos hacer que el modelo estándar se vuelva consistente bajo el monopolo magnético U(1) unido a algunos quarks o leptones conocidos?

Si la respuesta es no, ¿es esa la razón por la que no se debe continuar debido a (1) el hecho de la cancelación de la anomalía de gravedad de calibre en el modelo estándar, como una versión simple del mecanismo de Green-Schwarz para el modelo estándar ? ¿O el no-go se debe al hecho de que el monopolo magnético U(1) es en realidad una carga topológica, por lo que no puede ser invariante localmente y no puede ser creado por un operador local ? ¿O algo mas?

La cancelación de anomalía conocida para el modelo estándar se describe en Peskin y Schroeder Cap 20.2:ingrese la descripción de la imagen aquí

Ya doy mi propia respuesta dentro de mi pregunta, pero me gusta ver cómo la gente la entiende.
Sin embargo, es difícil creer que no los incluirán en el SM, ¡incluso si se encontraran por alguna razón de formalismo! Probablemente habrá que reajustar toda la teoría para incluir el mm.

Respuestas (2)

El modelo estándar generalmente se formula como una teoría de calibre con un Lagrangiano que contiene los campos de calibre. Digamos, por simplicidad, que consideramos una teoría de norma abeliana, lo que significa que el grupo de norma es tu ( 1 ) .

Entonces tienes un campo eléctrico y otro magnético, pero como sabes, estos se describen con el campo de fotones. A m . En términos de los campos eléctrico y magnético, A m corresponde a los potenciales. En particular, la parte espacial de A m es el vector A definido por B = A . Pero nótese que esta definición depende crucialmente del hecho de que B = 0 .

En presencia de monopolos magnéticos, esta relación se modifica para B = ρ metro dónde ρ metro es la densidad de cargas magnéticas, similar a la relación eléctrica mi = ρ mi . En este caso, ya no es cierto que exista una función A tal que B = A . Por supuesto, si tiene monopolos magnéticos pero no monopolos eléctricos (es decir, partículas cargadas bajo el campo eléctrico), entonces la relación mi = 0 le permite introducir una función A D tal que mi = A D , y puede proceder como de costumbre con mi y B intercambiado

En resumen, no podemos tener una descripción lagrangiana con partículas cargadas tanto eléctrica como magnéticamente. Esto generalmente se formula diciendo que las partículas cargadas eléctricamente y los monopolos magnéticos son mutuamente no locales. Y esta es la razón principal por la que no agregamos simplemente monopolos al modelo estándar de Lagrangian.

Como nota final, debo dar una pista de cómo se incluiría un monopolo magnético en el formalismo. Básicamente, se reduciría a una condición de contorno no trivial para el campo de calibre. Más precisamente, si desea describir un monopolo magnético que se encuentra en el punto X , luego quita una pequeña esfera alrededor X del espacio-tiempo y prescribir condiciones de contorno apropiadas para el campo A m sobre esta esfera. De esta manera, el monopolo no está realmente en el espacio-tiempo y desaparece la contradicción que describí anteriormente.

+1, gracias por el buen comentario. por cierto, pero también sabemos que hay objetos llamados dions que contienen carga eléctrica y monopolo magnético. Y también sabemos que el efecto de Witten en el término theta distinto de cero da lugar a dyones como partículas fundamentales. Me pregunto si su comentario también es cierto en este entorno. ¡Gracias de nuevo!
Sí, el comentario sigue siendo cierto, porque lo único que importa es que a bajas energías tienes un tu ( 1 ) teoría. Este tu ( 1 ) puede ser el grupo de calibre original que tenía a altas energías (en este caso, las partículas de baja energía son eléctricas), o algún otro tu ( 1 ) .
Creo que tienes en mente la solución de Seiberg-Witten para el puro norte = 2 teoría supersimétrica con grupo calibre S tu ( 2 ) , que es de hecho un buen patio de recreo. En este caso, el norte = 1 La teoría obtenida al dar una masa al multiplete quiral adjunto tiene dos vacíos distintos. En uno de ellos, los monopolos magnéticos se vuelven sin masa y en el otro, los diones se vuelven sin masa. Esto significa que a baja energía tenemos una tu ( 1 ) teoría con esos grados de libertad, y los dos vacíos están relacionados por un cambio de la θ -ángulo por 2 π .

Ninguna partícula elemental (en el sentido de una partícula que es el cuanto de un campo cuántico elemental) puede llevar una "carga magnética". La presencia de un campo "magnéticamente cargado" bajo el tu ( 1 ) es simplemente incompatible con el d F = 0 identidad de Bianchi, que sería modificada para d F = j metro para j metro la corriente magnética de 3. Esto significa que una teoría con campos elementales cargados magnéticamente no puede ser una simple tu ( 1 ) teoría del calibre. Discuto varias formas alternativas de incluir cargas magnéticas en un tu ( 1 ) R teoría aquí con un poco más de detalle.

La inexistencia de monopolos en el Modelo Estándar no tiene nada que ver con anomalías o cancelación de anomalías. El teorema de no-go para los monopolos topológicos en la teoría estándar (Glashow-Weinberg-Salam) de la interacción electrodébil es que los monopolos 't Hooft-Polyakov de cuatro dimensiones de un grupo de calibre GRAMO dividido en un subgrupo H requerir no trivial π 2 ( GRAMO / H ) del colector de vacío GRAMO / H . Si GRAMO es simple y doblemente conexo, es decir π 1 ( GRAMO ) = π 2 ( GRAMO ) = 0 , entonces esto es lo mismo que requerir no trivial π 1 ( H ) de la secuencia exacta larga de grupos de homotopía aplicada a la fibración H GRAMO GRAMO / H . Pero en la teoría GWS, el electromagnético H = tu ( 1 ) suele ser denso en el toro tu ( 1 ) × tu ( 1 ) S tu ( 2 ) × tu ( 1 ) para valores genéricos (es decir, irracionales) del ángulo de Weinberg, cf. las respuestas de Qmechanic aquí y aquí , convirtiéndose efectivamente en un no compacto tu ( 1 ) - el mapa de incrustación no es continuo en el nivel de los grupos de Lie.

Tenga en cuenta que esto solo excluye los monopolos 't Hooft-Polyakov. No excluye otros monopolos topológicos que podrían surgir de la topología no trivial del propio espacio-tiempo, como el ejemplo estándar de un tu ( 1 ) monopolo sentado en el origen de un ( R 3 / { 0 } ) × R , véase, por ejemplo, esta pregunta y respuesta .

Puedo preguntar: ¿No fue el modelo de inflación un esfuerzo, al principio, para explicar la ausencia de monopolos magnéticos? Corrígeme si me equivoco; ¿Existe una conexión entre ese esfuerzo y su respuesta? Quiero decir, ¿era algo que tenían en mente?
@ConstantineBlack ¿A cuál de los muchos modelos de inflación te refieres?
@ConstantineBlack El modelo no prohíbe los monopolos magnéticos, la inflación simplemente los diluye hasta el punto de que prácticamente no existen, cf. esta pregunta
+1, gracias por el buen comentario. por cierto, pero también sabemos que hay objetos llamados dions que contienen carga eléctrica y monopolo magnético. Y también sabemos que el efecto de Witten en el término theta distinto de cero da lugar a dyones como partículas fundamentales. Me pregunto si su comentario también es cierto en este entorno. ¡Gracias de nuevo!
@wonderich Hasta donde yo sé, los dyons son solo variantes de los monopolos 't Hooft-Polyakov que tienen carga magnética y eléctrica.